محاسبه چند دنباله

مدیران انجمن: parse, javad123javad

ارسال پست
نمایه کاربر
rohamavation

نام: roham hesami radرهام حسامی راد

محل اقامت: 100 مایلی شمال لندن جاده آیلستون، لستر، لسترشر. LE2

عضویت : سه‌شنبه ۱۳۹۹/۸/۲۰ - ۰۸:۳۴


پست: 3265

سپاس: 5494

جنسیت:

تماس:

محاسبه چند دنباله

پست توسط rohamavation »

$\sin x + \sin 2x + ... + \sin nx = \frac {\sin (\frac {n + 1} {2} x)} {\sin \frac{x}{2}} \sin \frac{nx}{2}$ که $\sum\limits_{k=1}^{n}sin(kx)$
روش اول اما از $\sin 2y = 2\sin y\cos y$ استفاده کنید
، با توجه به سمت چپ شما دریافت می کنیم:
$\frac {\sin y} {\sin \frac{x}{2}} \sin (y - \frac{x}{2}) + \sin2y =
\frac {\sin y} {\sin \frac{x}{2}} \sin (y - \frac{x}{2}) + 2\sin y\cos y =$
$\left(\frac {\sin (y-\frac{x}{2})} {\sin \frac{x}{2}} +2\cos y\right)\sin y = \left(\frac {\sin (y-\frac{x}{2}) +2\cos y\sin\frac{x}{2}} {\sin \frac{x}{2}}\right)\sin y =$
حالا از تفریق زوایای فرمول $sin(α-β)$ استفاده کنید.
$=\left(\frac {\sin y\cos \frac{x}{2} - \cos y \sin \frac{x}{2} +2\cos y\sin\frac{x}{2}} {\sin \frac{x}{2}}\right)\sin y = \left(\frac {\sin y\cos \frac{x}{2} +\cos y\sin\frac{x}{2}} {\sin \frac{x}{2}}\right)\sin y =$
و در آخر از افزودن زاویه فرمول sin(α+β) استفاده کنید
$=\left(\frac {\sin (y+ \frac{x}{2}) } {\sin \frac{x}{2}}\right)\sin y$
همان چیزی است که در سمت راست خود دارین
شما همچنین می توانیداینو ببینید
$\begin{align}
\sum_{k=0}^{n}\sin kx & =\Im \sum_{k=0}^{n}\mathrm{e}^{\mathrm{i}kx}\\
& = \Im \sum_{k=0}^{n}\left(\mathrm{e}^{\mathrm{i}x}\right)^k\\
& = \Im\frac{\left(\mathrm{e}^{\mathrm{i}x}\right)^{n+1}-1}{\mathrm{e}^{\mathrm{i}x}-1}\\
& = \Im\frac{\mathrm{e}^{\mathrm{i}(n+1)x}-1}{\mathrm{e}^{\mathrm{i}x}-1}\\
& = \Im\frac{\mathrm{e}^{\mathrm{i}\frac{n+1}{2}x}}{\mathrm{e}^{\mathrm{i}\frac{x}{2}}}\frac{\mathrm{e}^{\mathrm{i}\frac{n+1}{2}x}-\mathrm{e}^{-\mathrm{i}\frac{n+1}{2}x}}{\mathrm{e}^{\mathrm{i}\frac{x}{2}}-\mathrm{e}^{-\mathrm{i}\frac{x}{2}}}\\
& = \Im\mathrm{e}^{\mathrm{i}\frac{nx}{2}}\frac{\sin\left(\frac{n+1}{2}x\right)}{\sin\left(\frac{x}{2}\right)}\\
& = \sin\left(\frac{nx}{2}\right)\frac{\sin\left(\frac{n+1}{2}x\right)}{\sin\left(\frac{x}{2}\right)}.
\end{align}$
برای اثبات پست قبل اشاره ای کنم .قرار میدم
$s_n=sina+sin(a+d)+sin(a+3d)+...+sin(a+(n-1)d)$
دوطرف را در $2sin \frac{d}{2}$
ضرب می کنیم$2sin \frac{d}{2} s_n=2sin \frac{d}{2} sina+2sin \frac{d}{2}sin(a+d)+...+2sin \frac{d}{2} sin(a+(n-1)d)=cos(a- \frac{d}{2} ) -cos(a+ \frac{d}{2} )+cos(a+ \frac{d}{2} )-cos(a+ \frac{3d}{2} )+...+cos(a+ \frac{(2n-3)d}{2})-cos(a+ \frac{(2n-1)d}{2} )=cos(a- \frac{d}{2} )-cos(a+ \frac{(2n-1)d}{2})=$
با ادامه دادن این روش و حذف جملات غیر هم علامت و تبدیل جمع کسینوس ها به ضرب سینوسها به نتیجه ممکن میرسم
دقت کن $sin a+sin(a+d)+sin(a+3d)+....+sin(a+(n-1)d)= \frac{sin \frac{nd}{2} }{sin \frac{d}{2} } \times sin \frac{(2a+(n-1)d)}{2}$حال اگر قرار دهیم a=xوd=xداریم
$sinx+sin2x+...+sin (nx)= \frac{sin \frac{nx}{2} }{sin \frac{x}{2} } \times sin \frac{(n+1)x}{2}$
خوب اگه اینطور بیاد $\sin(x)+\frac{1}{2}\sin(2x)+\frac{1}{3}\sin(3x)+\frac{1}{4}\sin(4x)+...$
با استقرا ثابت کنید: $\sin(x)+\dfrac{1}{2}\sin(2x)+\dfrac{1}{3}\sin(3x)+\dfrac{1}{4}\sin(4x)+...$
"جمع بندی" سری $S=\sin(x)+\dfrac{1}{2}\sin(2x)+\dfrac{1}{3}\sin(3x)+\dfrac{1}{4}\sin(4x)+...$
$C=\cos(x)+\dfrac{1}{2}\cos(2x)+\dfrac{1}{3}\cos(3x)+\dfrac{1}{4}\cos(4x)+...$
$C+iS = e^{ix}+\dfrac{1}{2}(e^{ix})^2+\dfrac{1}{3}(e^{ix})^3+\dfrac{1}{4}(e^{ix})^4+...$
اجازه دهیدمن $e^{ix}$
سپس یک سری $t+\dfrac{t}{2}+\dfrac{t}{3}+\dfrac{t}{4}+...=\log(1+t)$ داریم.
که $-\log(1-e^{ix})=\log(1-\cos(x)-i\sin(x))$ است
از فرمول$\log(A+iB)=\dfrac{1}{2}\log(A^2+B^2)+\arctan\left(\dfrac{B}{A}\right)$ استفاده کنید
$-\log([1-\cos(x)]-i\sin(x))=-\dfrac{1}{2}\log([1-\cos(x)]^2-i\sin^2(x))-i\arctan\left(-\dfrac{\sin(x)}{1-\cos(x)}\right))$
. از آنجایی که ما فقط به بخش خیالی علاقه داریم، جمع S را داریم است:$-\arctan\left(-\dfrac{\sin(x)}{1-\cos(x)}\right)$
که $-\arctan\left(-\cot(\dfrac{x}{2})\right)$
نمیدونم بعدش چیکار کنم مجموع این سری به گفته اویلر باید$\dfrac{\pi-x}{2}$باشد
یک راه به نظرم رسیداول، سری هستند
$\log(1+x)=\sum_{k=1}^\infty(-1)^{k-1}\frac{x^k}k$
و$-\log(1-x)=\sum_{k=1}^\infty\frac{x^k}k$
سری $2\pi$ است دوره ای$-\pi\le x\le\pi$ و مجموع $\begin{align}
\sum_{k=1}^\infty\frac{\sin(kx)}k
&=\frac1{2i}\sum_{k=1}^\infty\frac{e^{ikx}-e^{-ikx}}k\\
&=-\frac1{2i}\left(\log(1-e^{ix})-\log(1-e^{-ix})\right)\\
&=\frac1{2i}\log\left(\frac{1-e^{-ix}}{1-e^{ix}}\right)\\
&=\frac1{2i}\log\left(-e^{-ix}\right)\\[3pt]
&=\operatorname{sgn}(x)\frac\pi2-\frac x2
\end{align}$
است میدونی که sgn تابع علامتیا تابع پله ‌ای Step Function هست ${\displaystyle \operatorname {sgn} x:={\begin{cases}-1&{\text{if }}x<0,\\0&{\text{if }}x=0,\\1&{\text{if }}x>0.\end{cases}}}$که تنها سه مقدار صفر، ۱و ۱- را به عنوان خروجی برمی‌گردونه. این تابع به ازای مقادیر مثبت عدد ۱ ، به ازای مقدار صفر عدد صفر و به ازای مقادیر منفی عدد ۱- را به عنوان خروجی برمی‌گردونه
سوال بعدی مجموع $\sum_{k=1}^{n}\sin^{2}kx$
نمایش اون راحته
$\sin A-\sin B=2\cos\left(\frac{A+B}{2}\right)\sin\left(\frac{A-B}{2}\right),$
$\sin\left(2k+1\right)x-\sin\left(2k-1\right)x=2\cos2kx\cdot \sin x.$
از این می توانیم نشان دهیم
$\sum_{k=1}^{n}\sin^{2}kx=\frac{n}{2}-\frac{\sin\left(nx\right)\cdot \cos\left(n+1\right)x}{2\sin x}$
با روش تفاوت؟توالی و سلسله مثلثات
فرمول زاویه دوگانه $\sin^2(kx) = \frac{1 - \cos(2kx)}{2}$را به ما میده
پس ما داریم
$\sum\limits_{k = 1}^n\sin^2(kx) = \sum\limits_{k = 1}^n\frac{1 - \cos(2kx)}{2} = \frac{n}{2} - \frac{1}{2}\sum\limits_{k = 1}^n\cos(2kx)$
$= \frac{n}{2} - \frac{1}{4\sin x}\sum\limits_{k = 1}^n2\cos(2kx)\sin x$
$= \frac{n}{2} - \frac{1}{4\sin x}\sum\limits_{k = 1}^n\left[\sin(2k+1)x-\sin(2k-1)x\right]$
$= \frac{n}{2} - \frac{1}{4\sin x}\left[\sin(2n+1)x - \sin x\right]$
$= \frac{n}{2} - \frac{1}{4\sin x}\left[2\cos(n+1)x\sin nx\right]$
$= \frac{n}{2} - \frac{\cos(n+1)x\sin nx}{2\sin x}$ارائه sinx≠0، یعنی x مضرب صحیح $\pi$ نیست
. اگر چه در این موارد اگر به عنوان حد در نظر گرفته شود، برابری همچنان صادق است.
شما $\sum\limits_{k=1}^{n}sin(x^k)$ رو حساب کنید برای شما هوپاییها گذاشتم
خوب منم یکی دیگه میارم اثبات اینکه دنباله $F_{n}(x)=\sum\limits_{k=1}^{n} \frac{\sin{kx}}{k}$
به طور مرزی روی R همگرا استابتدا، بیایید توجه داشته باشیم که برای $x\in(0,2\pi)$
$\sum\limits_{k=1}^{n}\frac{\sin kx}{k}=\int_{0}^{x}\sum\limits_{k=1}^{n}\cos kt\ dt=
-\frac{x}{2}+\int_{0}^{x}\frac{\sin \frac{(2n+1)t}{2}}{2\sin{\frac{t}{2}}}\ dt$
$=-\frac{x}{2}+\int_{0}^{x}\left(\frac{1}{2\sin{\frac{t}{2}}}-\frac{1}{t}\right)\sin \frac{(2n+1)t}{2}\ dt +\int_{0}^{x}\frac{\sin \frac{(2n+1)t}{2}}{t}dt.$
خوب ، انتگرال اول در سمت راست توسط لم ریمان-لبگ به صفر میل میکنه. دومی برابر است از روش یک جایگزینی $s=(2n+1)t/2$
) انتگرال
$\int_{0}^{\frac{(2n+1)x}{2}}\frac{\sin s}{s}\ ds\to\int_{0}^{\infty}\frac{\sin s}{s}\ ds=\frac{\pi}{2}.$
از این رو $\lim\limits_{n\to\infty}\ \sum\limits_{k=1}^{n}\frac{\sin kx}{k}=\frac{\pi-x}{2}=f(x),\qquad x\in(0,2\pi).$
سریال روی R همگرا می شود
روش دوم برای هر n≥1
، می دانیم نقاط ثابت $F_n(x)$ کجا هستند از زمان$F_n'(x)$ انجام میشه دارای فرم بسته ساده است.
نتیجه می شود که
$\sup_{x\in\mathbb{R}} |F_n(x)| = \sum_{k=1}^{n}\frac{1}{k}\sin\left(\frac{2\pi k}{2n+1}\right)$
و بیشتر از $[0,\pi]$اصطلاح بزرگترین کران پایین (Inf) و کوچکترین کران بالا (Sup) مجموعه را با Infimum یا به اختصار Inf نشان می‌دهند. منظور از بزرگترین کران پایین یک مجموعه، بزرگترین مقداری است که از همه عناصر مجموعه کوچکتر است. این امر به مانند مقدار کمینه یا Minimum در یک مجموعه است با این تفاوت که مقدار کمینه یک مجموعه باید عضوی از آن مجموعه باشد ولی در مورد Inf چنین شرطی وجود نداره
بنابراین چون Inf ممکن است عضوی از مجموعه مورد نظر نباشد، آن را به عنوان یک کران در نظر می‌گیرند و بزرگترین کران پایین هست.
از طرفی منظور از کوچکترین کران بالا یک مجموعه که با Supremum یا به اختصار Sup نشان داده می‌شود، کوچکترین مقداری است که از همه اعضای مجموعه بزرگتر است. Sup درست مانند مقدار بیشنیه یا Maximum است، با این تفاوت که مقدار بیشینه باید عضوی از مجموعه باشد ولی در مورد Sup چنین شرطی وجود ندارد.ببین $\large LB(S)=\{a\in T;\; a\leq x \forall x \in S\}$به این ترتیب مقادیری مثل a، کران پایین مجموعهS را تشکیل می‌دهند. واضح است که a در مجموعهTقرار دارد.بزرگترین کران پایین یا Inf مجموعه
S را هم به صورت زیر تعریف میکنم.$\large a \text{ is inf};\;\; \forall y \in LB(S)\subseteq T, y\leq a$
این امر به این معنی است که در بین همه کران‌های پایین مجموعه S، مقدار a از همه بزرگتر است. در این صورت میارم $\large \inf(S)=a$کوچکترین کران بالا (Sup)تعریف کوچکترین کران بالا (Sup) نیز درست به مانند Inf صورت می‌گیره مجموعه مرتب جزئی (T,≤) را در نظر بگیرید. برای زیر مجموعه S از آن، کران پایین به صورت زیر تعریف می‌شود.$\large GB(S)=\{b\in T;\; x\leq b \forall x \in S\}$بر همین اساس و به کمک مجموعه GB، کوچکترین کران بالا را به عنوان Sup‌ مجموعه S در نظر می‌گیریم. به عبارت دیگر کوچکترین عضو مجموعه GB همان Sup مجموعه S هست $\large b \text{ is sup};\;\; \forall z \in GB(S)\subseteq T, b \leq z$
ما $\sin(x)\leq \frac{4}{\pi^2}x(\pi-x)$ داریم بنابراین توسط فرورفتگی $\sup_{x\in\mathbb{R}} |F_n(x)| \leq \frac{8n^2}{(2n+1)^2} <2.$ما همچنین ممکن است ثابت کنیم که دنباله$A_n = \sum_{k=1}^{n}\frac{1}{k}\sin\left(\frac{2\pi k}{2n+1}\right)$در حال افزایش و همگرا به$\int_{0}^{\pi}\frac{\sin x}{x}\,dx = \text{Si}(\pi) \approx 1.85194.$
∫π0sinxxdx=Si(π)≈1.85194.
در نهایت ممکن است بررسی کنیم که $\sum_{k\geq 1}\frac{\sin(kx)}{k}$
سری فوریه موج دندانه اره است، یعنی $2\pi$گسترش دوره ای $\frac{\pi-x}{2}$ تعریف شده بیش از $(0,2\pi)$
. این برای اطمینان از همگرایی در $L^2$کافیه به علاوه یک کران یکنواخت برای |Fn(x)| داریم
.من دیروز یک روش از راه حل این مشکل به ذهنم رسید $\sum_{k=1}^{n} \sin (kx)$ و همه آن را درک می کنم اما می خواستم تأیید کنم که آیا قرار است بتوانم آن را از این طریق حل کنم یا خیر. راه حل اینجاست.
$\sin(x) = \frac{e^{ix}-e^{-ix}}{2i}$
$\sin(kx) = \frac{e^{ikx}-e^{-ikx}}{2i}$
$\sum_{k=1}^{n} \sin(kx) = \frac{1}{2i}\left(\sum_{k=1}^{n} e^{ikx}-\sum_{k=1}^{n} e^{-ikx}\right)$
$= \frac{1}{2i} \left(\sum_{k=1}^{n} \left(e^{ix}\right) ^ {k}-\sum_{k=1}^{n}\left(e^{-i x}\right)^{k}\right)$
نکته: $\text { Note: } \sum_{k=1}^{n} x^{k}=\frac{x-x^{n+1}}{1-x}$
$\sum_{k=1}^{n} \sin (kx)=\frac{1}{2i}\left(\frac{e^{ix}-e^{(n+1)ix}}{1-e^{i x}}-\frac{e^{-ix}-e^{-(n+1)ix}}{1-e^{-ix}}\right)$
$=\frac{1}{2i}\left(\frac{e^{i x}\left(1-e^{i n x}\right)}{e^{\frac{ix}{2}}\left(e^{-\frac{ix}{2}}-e^{\frac{ix}{2}}\right)}-\frac{e^{-i x}\left(e^{-i x}-e^{-i m x}\right)}{e^{-\frac{ix}{2}}\left(e^{\frac{ix}{2}}-e^{-\frac{ix}{2}}\right)}\right)$
$=\frac{1}{2i}\left(\frac{e^{\frac{ix}{2}}\left(1-e^{inx}\right)}{-2i \sin \left(\frac{x}{2}\right)}-\frac{e^{-\frac{ix}{2}}\left(1-e^{-inx}\right)}{2i \sin \left(\frac{x}{2}\right)}\right)$⎠
$=\frac{1}{4} \cdot \frac{e^{\frac{ix}{2}}-e^{ix\left(n+\frac{1}{2}\right)}+e^{-\frac{ix}{2}}-e^{-ix\left(n+\frac{1}{2}\right)}}{\sin \left(\frac{x}{2}\right)}$
$=\frac{1}{4} \cdot \frac{e^{\frac{ix}{2}}+e^{-\frac{ix}{2}}-e^{ix\left(n+\frac{1}{2}\right)}-e^{-ix\left(n+\frac{1}{2}\right)}}{\sin \left(\frac{x}{2}\right)}$
$=\frac{1}{4} \cdot \frac{2\cos \left(\frac{x}{2}\right) -2 \cos \left(x\left(n+\frac{1}{2}\right)\right)}{\sin \left(\frac{x}{2}\right)}$
$=\frac{1}{2} \cdot \frac{\cos \left(\frac{x}{2}\right)-\cos \left(nx+\frac{x}{2}\right)}{\sin \left(\frac{x}{2}\right)}$
سوال من این است که از آنجایی که اگر نسبت هندسی 1 باشه فرمول جمع هندسی کار نمیکنه، آیا می توانم از آن در این راه حل به عنوان$\mid e^{ix}\mid$استفاده کنم.
همیشه 1 است؟ یا راه حل دیگری برای این سوال وجود دارد؟
.در اینجا یک پاسخ ساده تر است.با استفاده از فرمول حاصل از جمع، داریم
$\begin{aligned}
2 \sin \frac{x}{2}\left(\sum_{k=1}^{n} \sin k x\right) & =\left(\cos \frac{x}{2}-\cos \frac{3}{2}x\right)+\left(\cos \frac{3}{2}x-\cos \frac{5}{2}x\right)+\cdots \\
& +\left[\cos \left(n-\frac{1}{2}\right) x-\cos \left(n+\frac{1}{2}\right) x\right] \\
& =\cos \frac{x}{2}-\cos\left(n+\frac{1}{2}\right) x.
\end{aligned}$.
وقتی $\sin \frac{x}{2} \neq 0$
، ما داریم$\sum_{k=1}^{n} \sin k x=\frac{\cos \frac{x}{2}-\cos\left(n+\frac{1}{2}\right) x}{2 \sin \frac{x}{2}}.$
به همین ترتیب،$\begin{aligned}
2 \sin \frac{x}{2}\left(\frac{1}{2}+\sum_{k=1}^{n} \cos k x\right) & =\sin \frac{x}{2}+\left(\sin \frac{3}{2} x-\sin \frac{x}{2}\right)+\cdots \\
& +\left[\sin \left(n+\frac{1}{2}\right) x-\sin \left(n-\frac{1}{2}\right) x\right] \\
& =\sin \left(n+\frac{1}{2}\right) x.
\end{aligned}$
وقتی $\sin \frac{x}{2} \neq 0$
، ما داریم$\frac{1}{2}+\sum_{k=1}^{n} \cos k x=\frac{\sin \left(n+\frac{1}{2}\right) x}{2 \sin \frac{x}{2}}.$
در ضمن ابجی من یک ماشین NISSAN LEAF مدل2020 که 14000 مایل کارکرده هیبریدی قرمز متالیک گرفت قیمت £18,290 وقسطی اونم 60 ماهه تا پایان مدت ویزا و تازه ماهی حدود 300 پوند با درامد میخونه اینکه حهت رانندگی اونجا از چپ هست واقعا سخته صدور گواهینامه توسط DVLA صادر میشه نظر خودم فورد بود اونم رفتیم از لندن گرفتیم این بار فقط چند خیابون لندن دیدم
تصویر

ارسال پست